Categories
Exam Questions Harvard

Harvard. Exam questions for “The Labor Question in Europe and the U.S.” Edward Cummings, 1894

 

Edward Cummings, the father of the poet E. E. Cummings, covered the social economics course offerings at Harvard at the end of the 19th century. These included courses in labor economics and social reform movements (esp. “socialism and communism”). The exams for his comparative labor course from 1893-94 mostly consisted of specific unidentified quotations, with students asked to provide explanations or comments. The text-search functions at hathitrust.org, archive.org and google located a dozen of the quotations used by Cummings and links to the texts have been provided below.

___________________________

Course Description
(from 1896-97)

The Labor Question in Europe and the United States. — The Social and Economic Condition of Workingmen. Tu., Th., Sat., at 10. Asst. Professor Edward Cummings and Dr. John Cummings.    (VIII)

Course 9 is a comparative study of the condition and environments of workingmen in the United States and European countries. It is chiefly concerned with problems growing out of the relations of labor and capital. There is careful study of the voluntarily organizations of labor, — trade unions, friendly societies, and the various forms of cooperation; of profit-sharing, sliding scales, and joint standing committees for the settlement of disputes ; of factory legislation, employers’ liability, the legal status of laborers and labor organizations, state courts of arbitration, and compulsory government insurance against the exigencies of sickness, accident, and old age. All these expedients, together with the phenomena of international migration, the questions of a shorter working day and convict labor, are discussed in the light of experience and of economic theory, with a view to determining the merits, defects, and possibilities of existing movements.

The descriptive and theoretical aspects of the course are supplemented by statistical evidence in regard to wages, prices, standards of living, and the social condition of labor in different countries.

Topics will be assigned for special investigation, and students will be expected to participate in the discussion of selections from authors recommended for a systematic course of reading.

The course is open not only for students who have taken Course 1, but to Juniors and Seniors of good rank who are taking Course 1.

 

Source: Harvard University, Faculty of Arts and Sciences. Division of History and Political Science Comprising the Departments of History and Government and Economics, 1897-98, pp. 36-37.

___________________________

Course Enrollment
1893-94

[Economics] 9. Asst. Professor Cummings. — The Social and Economic Condition of Workingmen in the United States and in other countries. 3 hours.

Total 43: 7 Graduates, 16 Seniors, 11 Juniors, 3 Sophomores, 1 Freshman, 5 Other.

Source: Harvard University. Annual Report of the President of Harvard College, 1893-94, p. 61.

___________________________

Mid-year Examination
ECONOMICS 9
1893-94.

(Arrange your answers in the order in which the questions stand. So far as possible illustrate your discussions by a comparison of the experience of different countries. Omit two questions.)

  1. “It becomes my duty, therefore, in undertaking to interpret the social movement of our own times, to disclose, first, those changes in industrial methods by which harmony in industries has been disturbed, and then to trace the influence of such changes into the structure of society.” State carefully what these changes have been; and trace their influence.
    [Henry C. Adams. “An Interpretation of the Social Movements of our Time”, International Journal of Ethics, Vol II, October, 1891), p. 33]
  2. Discuss the effect upon wages of machinery, — (a) as a substitute for labor (b) as auxiliary to labor; (c) as affecting division of labor; (d) as concentrating labor and capital; (e) as affecting the nobility[sic, “mobility”] of labor and capital.
  3. “In my opinion, combination among workingmen is a necessary step in the re-crystallization of industrial rights and duties.” State fully your reasons for agreeing or disagreeing with this opinion. What forms of combination do you include?
    [Henry C. Adams. “An Interpretation of the Social Movements of our Time”, International Journal of Ethics, Vol II, October, 1891), p. 45]
  4. “Trade-unions have been stronger in England than on the Continent, and in America….” In what respects stronger? Why? Contrast briefly the history and present tendencies of the trade-union movement in the United States, England, France, Germany, and Italy.
    [Alfred Marshall, Elements of Economics of Industry: being the First Volume of Elements of Economics (London: Macmillan, 1892), Book VI, Ch. XIII. §18, p. 404]
  5. “Trade-unions have been stronger in England than on the Continent, and in America; and wages have been higher in England than on the Continent, but lower than in America.” “Again, those occupations in which wages have risen most in England happen to be those in which there are no unions.” How far do such facts impeach the effectiveness of trade-unions as a means of raising wages and improving the condition of workingmen? What do you conceive to be the economic limits and the proper sphere of trade-union action?
    [Alfred Marshall, Elements of Economics of Industry: being the First Volume of Elements of Economics (London: Macmillan, 1892), Book VI, Ch. §18, pp. 404-405.]
  6. “We saw at the beginning that in comparatively recent years the difficulties of keeping up a purely offensive and defensive organization had brought many of the unions back nearer their old allies, the friendly societies, and emphasized the friendly benefits in proportion as the expenditure for trade disputes seemed less important.” Explain carefully this earlier and later relation of trade-unions and Friendly Societies in England.
    [Edward Cummings, The English Trades-Unions, Quarterly Journal of Economics, Vol. III (July, 1889), p. 432.]
  7. “This spirit of independent self-help has its advantages and its disadvantages. We have already had occasion to remark how slow in these Friendly Societies has been the progress of reform, and we must repeat that up to the present day it still exhibits defects.” Explain and illustrate the progress of the reform and the nature of existing defects. Does English self-help experience suggest the desirability or undesirability of imitating German methods of compulsory insurance?
  8. “Countless[sic, “Doubtless” in original] boards of arbitration and conciliation, the establishment of certain rules of procedure, agreements covering definite periods of time, may aid somewhat in averting causes of dispute or in adjusting disputes as they arise; but if we have these alone to look to, strife will be the rule rather than the exception.” Explain the various methods adopted and the results obtained. What have you to say of “compulsory arbitration?”
    [Francis A. Walker. “What Shall We Tell the Working Classes?” Scribner’s Magazine, Vol. 2, 1887.  Reprinted in Discussions in Economics and Statistics, edited by Davis R. Dewey. Vol. II. 315-316.]
  9. “The conclusion of the whole matter seems to be, that what is desirable is not so much to put a stop to sub-contracting as to put a stop to ‘sweating,’ whether the man who treats the workman in the oppressive manner which the word ‘sweating’ denotes be a sub-contractor, a piece-master, or a contractor.” Indicate briefly some of the principal forms of industrial remuneration, — giving the special merits and defects of each.
    [David F. Schloss. Methods of Industrial Remuneration (London: Williams and Norgate, 1892), p. 140.]
  10. “Now that I am on piece-work, I am making about double what I used to make when on day-work. I know I am doing wrong. I am taking away the work of another man.” State and criticize the theory involved in this view of production.
    [David F. Schloss. Methods of Industrial Remuneration (London: Williams and Norgate, 1892), p. 43-44.]

 

Source: Harvard University Archives. Harvard University, Mid-year examinations, 1852-1943. Box 3, Volume: Examination Papers, Mid-Year, 1893-94.

___________________________

Year-End Examination
ECONOMICS 9.
1893-94.

(Arrange your answers in the order in which the questions stand. So far as possible illustrate your discussions by a comparison of the experience of different countries. Take the first three questions and four others.)

  1. “As soon, however, as the factory system was established, the inequality of women and children in their struggle with employers attracted the attention of even the most careless observers; and, attention once drawn to this circumstance, it was not long before the inequality of adult men was also brought into prominence.” How far is this true (a) of England, (b) of the United States? Trace briefly the legislative consequences for children and for adults in the two countries.
    [Arnold Toynbee. Lectures on the Industrial Revolution of the 18th Century in England (The Humboldt Library of Popular Science Literature, Vol. 13. New York: Humboldt Publishing Co.), p. 17.]
  2. “It will be necessary, in the first place, to distinguish clearly between the failure of Industrial Coöperation and the failure of the coöperative method—a method, as we have seen, adopted, even partially, by only a very small fraction of Industrial Coöperation.” Explain carefully, discussing especially the evidence furnished by France and England.
  3. “These four concerns—the Maison Leclaire, the Godin Foundry, the Coöperative Paper Works of Angoulême and the Bon Marché—are virtually coöperative; certainly they secure to the employers and stockholders the substantial benefits of purely coöperative productive enterprises, while they are still, logically, profit-sharing establishments.” State your reasons for agreeing or disagreeing. Indicate briefly the characteristic features of each enterprise.
  4. “What inferences are we to draw from the foregoing statistics? Unmistakably this, that the higher daily wages in America do not mean a correspondingly enhanced labor cost to the manufacturer. But why so?” Discuss the character of available evidence in regard to the United States, Great Britain and the continent of Europe.
    [E. R. L. Gould. The Social Condition of Labor (Baltimore: Johns Hopkins Press, January 1893), pp. 41-2.]
  5. “The juxtaposition of figures portraying the social-economic status of workmen of different nationalities in the country of their birth and the land of their adoption furnishes lessons of even higher interest. From this we are able to learn the social effect of economic betterment.” Explain. How do the facts in question affect your attitude toward recent changes in the character and volume of our immigration?
    [E. R. L. Gould. The Social Condition of Labor (Baltimore: Johns Hopkins Press, January 1893), pp. 35-6.]
  6. “The Senate Finance Committee issued some time ago a comparative exhibit of prices and wages for fifty-two years, from which the conclusion is generally drawn that the condition of the wage earner is better to-day than it was thirty or forty years ago. A conclusion of this kind reveals the weakness of even the best statistics. No one can doubt that the work of the Finance Committee is work of high excellence, but for comparing the economic condition of workers it is of little value.” Do you agree or disagree? Why? Indicate briefly the character of the evidence.
  7. What are the principle organizations which may be said to represent the “Labor Movement” in the United States at the present time? How far are they helpful and how far hostile to one another?
  8. “In a preceding chapter I have said that as a moral force and as a system the factory system of industry is superior to the domestic system, which it supplanted.” State your reasons for agreeing or disagreeing.
    [Carroll D. Wright. Factory Legislation from Vol. II, Tenth Census of the United States, reprinted in First Annual Report of the Factory Inspectors of the State of New York (Albany, 1887), p. 41.]
  9. Contrast the English and the German policy in regard to Government Workingmen’s Insurance.
  10. “Gladly turning to more constructive work, I next consider some industrial changes and reforms which would tend to correct the present bias towards individualism.” What are they?
  11. Give an imaginary family budget for American, English and German operatives in one of the following industries, — coal, iron, steel, cotton, wool, glass, indicating roughly characteristic differences in such items as throw most light on the social condition of labor.

 

Source: Harvard University Archives. Harvard University, Examination papers, 1873-1915. Box 4, Volume: Examination Papers, 1893-95.pp. 39-41.

Image Source: University and their Sons. History, Influence and Characteristics of American Universities with Biographical Sketches and Portraits of Alumni and Recipients of Honorary Degrees. Editor-in-chief, General Joshua L. Chamberlain, LL.D. Vol II (1899), pp. 155-156.

 

Categories
Exam Questions Harvard Suggested Reading Syllabus

Harvard. Readings, midterm and final exams for economic growth course. Kuznets, 1960-61

 

Simon Kuznets (b. 1901; d. 1985) left Johns Hopkins University to join the Harvard economics faculty beginning with the 1960-61 academic year. This post provides the reading list and exams for Kuznets’ signature course on economic growth from his first year as a Harvard professor. 

_________________________

Course Enrollment

[Economics] 203 Economic Growth and Comparative Economic Structures. Professor Kuznets. Full course.

(F) Total 23: 12 Graduates, 1 Senior, 1 Junior, 2 Radcliffe, 7 Other Graduate.

(S) Total 22: 12 Graduates, 1 Senior, 1 Junior, 2 Radcliffe, 6 Other Graduates.

Source: Harvard University. Report of the President of Harvard College, 1960-1961, p. 77.

_________________________

HARVARD UNIVERSITY
Department of Economics
Economics 203

Long Term Changes and International Differences—National Income and its Components

GENERAL

  1. Simon Kuznets, “National Income and Industrial Structure,” in Economic Change, Chapter 6, 145-192.
  2. Simon Kuznets, “International Differences in Income Levels,” ibid., pp. 216-252.
  3. Colin Clark, Conditions of Economic Progress, 3rd Edition 1957 or any of the earlier editions (for browsing).
  4. M. Gilbert and I.B. Kravis, An International Comparison of National Products and the Purchasing Power of Currencies, Paris, O.E.E.C., 1954.
    or (Comparative National Products and Price Levels, 1958)

RATES OF GROWTH

  1. Simon Kuznets, Economic Development and Cultural Change, Vol. V, no. 1, October 1956.
  2. Simon Kuznets, Six Lectures on Economic Growth, Free Press, 1959, pp. 13-41.

INDUSTRIAL STRUCTURE

  1. A.G.B. Fisher, The Class of Progress and Security, London 1935.
  2. E.M. Ojala, Agriculture and Economic Progress, Oxford University Press, 1952.
  3. T.W. Schultz, The Economic Organization of Agriculture, N.Y. 1953, part I.
  4. Hollis B. Chenery, “Patterns of Industrial Growth,” American Economic Review, September 1960, pp. 624-654.
  5. P.T. Bauer and B.S. Yamey, Economic Journal, December 1951, pp. 741-755.
  6. Simon Kuznets, Economic Development and Cultural Change, (a) Supplement to No. 4, Vol. V, July 1957; (b) Part 2, Vol. VI, July 1958, also Six Lectures on Economic Growth, pp. 43-67.

FACTOR SHARES

  1. R. M. Solow, “The Constancy of Relative Shares,” American Economic Review, September 1952, pp. 618-30.
  2. I. B. Kravis, “Relative Income Shares in Fact and Theory,” American Economic Review, December 1959, pp. 917-947.
  3. Simon Kuznets, Economic Development and Cultural Change, Vol. III, No. 3, Part 2, April 1959.

CAPITAL FORMATION

  1. M. Abramovitz, ed., Capital Formation and Economic Growth, Princeton 1955. Papers by Kuznets, Goldsmith, Usher, MacLaurin, and Rostow.
  2. Simon Kuznets, Economic Development and Cultural Change,
    (a) Vol. VIII, no. 4, Part II, July 1960.
    (b) Vol. IX, no. 3, Part II, April 1961

CONSUMPTION PATTERNS

  1. M. K. Bennett, “International Disparities in Consumption Levels,” American Economic Review, September 1951, pp. 632-649.
  2. Simon Kuznets, Regional Economic Trends and Levels of Living,” in P.M. Hauser, Population and World Politics, Free Press 1958.
  3. International Association for Research in Income and Wealth, Income and Wealth Series II, Chapter VI, particularly pp. 167-177. (1953)

INCOME DISTRIBUTION BY SIZE

  1. I. B. Kravis, “International Differences in the Distribution of Income,” The Review of Economics and Statistics, November 1960, pp. 402-416.
  2. Simon Kuznets, “Economic Growth and Income Inequality,” American Economic Review, March 1955, pp. 1-28.

Source: Harvard University Archives. Syllabi, course outlines and reading lists in Economics, 1895-2003. Box 8, Folder “Economics, 1960-1961 (2 of 2)”.

_________________________

HARVARD UNIVERSITY
Department of Economics

Economics 203
Midyear Examination
January 1961

Choose 6 out of the 8 questions, omitting one out of group 2-5 and one out of group 6-8.

Please write clearly and concisely. An outline of the answer rather than a full presentation is also acceptable, provided the outline is sufficiently detailed and informative.

  1. Outline the time pattern of rates of natural increase of population in the transition from the pre-modern period to modern growth, distinguishing movements of birth rates and death rates, separately for the older (European) and younger (overseas) countries. Comment briefly on probable causes.
  2. State the connection between Malthus’ population theory, the theory of differential rent, the law of diminishing returns, the iron law of wages, and the theory of long-term trends in distribution of national product and of the approach to the stationary state.
  3. Outline the types of evidence claimed in support of Pearl’s “law of population growth” and comment on their validity.
  4. What is the general structure of assumptions underlying an empirical projection of population growth? Distinguish the theory of the model from the theory of deviations and comment briefly on each.
  5. Indicate the reasons adduced by Alvin Hansen to demonstrate the depressive effects on economic growth of retardation in the growth of population, and evaluate them.
  6. Outline the major features of trans-ocean trade in the centuries immediately prior to the industrial revolution, and the various contributions made by it to the emergence of modern capitalism.
  7. Why was the industrial revolution concentrated primarily in cotton, iron, steel, and steam power?
  8. Discuss the slackening in the rate of technical change in the cotton textile industry after the mid-nineteenth century, as well as in the economic effects of technical change in the industry. Indicate both the ways of measuring the changes, and the reasons for slackening in the rate of their occurrence.

Source: Harvard University Archives. Social Sciences, Final Examinations. Vol. 131. January 1961. Papers Printed for Final Examinations. History, History of Religions, …, Economics, …, Naval Science, Air Science. January 1961.

_________________________

HARVARD UNIVERSITY
Department of Economics

Economics 203
Final Examination
June 6, 1961

Answer at least 4 questions, choosing at least one from each of the four Roman numeral groups. You may answer more if you wish. A detailed outline of an answer, instead of a complete text, is acceptable.

Group I

  1. In what sense is the association between technical changes and scientific discoveries, illustrated by the history of the radio industry, different from that characterizing the inventions of the Industrial Revolution? Discuss.
  2. Outline (and discuss) factors making for a retardation in the rate of growth, observable in most specific industries in the Western European countries and in the United States.

Group II

  1. Indicate properties of national income or product estimates as measures of economic growth. Consider particularly limitations arising out of difficulties as to scope, netness, and valuation.
  2. What are the implications of the rates of growth in per capita product, observed for the past half century or longer for the developed countries, as to the comparative levels of per capita product in underdeveloped countries today and in the presently developed countries just prior to their industrialization? Discuss.

Group III

  1. Discuss the factors involved in the long-term decline, in the process of modern economic growth, in the share of agriculture in national product? In labor force? Distinguish clearly between the trends in the share in product and in the share in labor force.
  2. How do you explain the rise in the shares of labor force attached to the service industries in the course of economic growth? Define the service industries before answering the question.

Group IV

  1. Define various types of proportion of capital formation to aggregate product (gross-net, domestic-national, etc.) that can be calculated; the corresponding types of capital-output ratios; and discuss their possible use in the analysis of economic growth.
  2. Discuss factors that might have made for a rising trend in the proportion of capital formation to national product observed in many (if not all) countries. Distinguish between gross and net capital formation proportions; and between domestic and national.

Source: Harvard University Archives. Social Sciences, Final Examinations. Vol. 134. June 1961. Papers Printed for Final Examinations. History, History of Religions, …, Economics, …, Naval Science, Air Science. June 1961.

Image Source: National Academy of Sciences. 2001. Biographical Memoirs: Volume 79. Washington, DC: The National Academies Press, p. 202.

Categories
Chicago Exam Questions Fields History of Economics

Chicago. History of Economic Thought, Ph.D. preliminary exam. Summer, 1989

 

The previous post provided the transcribed questions for the 1974 version of the Chicago prelim exam for the history of economic thought. Here we have the questions for a fifteen year younger exam Presumably both these sibling exams were authored by George Stigler in whose archived papers they can be found.

______________________

History of Economic Thought Prelim Exam
Summer 1989

Answer Question 1 or Question 2, not both:

  1. Sam Hollander argues that David Ricardo’s Principles is really a neoclassical analysis (such as Marshall’s), although written in a different style and laying different amounts of emphasis upon various parts of the theory (for example, more emphasis on cost, less on demand).
    1. If this is true of Ricardo, why not also of Adam Smith? How do these two differ?
    2. What is neoclassical (Marshallian) or not neoclassical about Ricardo’s treatment of wages on average, or of wages in individual occupations?
  2. In his recent review of Samuel Hollander’s study of J. S. Mill, Pedro Schwartz argued that Hollander failed to see that J.S. Mill had a very different view of the scope of economics than Smith or Ricardo. Mill “treated (economics) as a limited science whose rationale is irreconcilable to the guiding principles of ethics and politics.”

From your knowledge of Mill’s Principles, defend Schwartz or Hollander.

Answer all of the remaining questions:

  1. Do people know what is good for them? Show how Smith and J.S. Mill draw their conclusions on this question.
  2. Arguments have often persisted for long periods over what an economist really meant. Ricardo is a favorite example, but there is hardly an economist of note who has escaped this sort of dispute. Compare the roles of…
    1. …a careful analysis of what the economist meant (relying on his writings, letters, etc.)…
    2. …a careful analysis of what his contemporaries and immediate successors thought he meant…

…in resolving such disputes. Which is the more important basis of judgment, and why? Apply both techniques to Malthus’ use of the arithmetic and geometric ratios.

  1. “Every individual is continually exerting himself to find out the most advantageous employment for whatever capital he can command…the study of his own advantage naturally, or rather necessarily leads him to prefer that employment which is most advantageous to the society.
    First, every individual endeavors to employ his capital as near home as he can, and consequently as much as he can in the support of domestic industry.
    Thus, upon equal or nearly equal profits, every wholesale merchant naturally prefers the home trade to the foreign trade. …In the home trade his capital is never so long out of his sight as it frequently is in the foreign trade…yet for the sake of having some part of his capital always under his own view and command, he willingly submits to this extraordinary charge (double charge of loading and unloading as well as to the payment of some duties and customs).”

In this passage, famous for arguing free trade, Smith seems to make a case (a) for preferring domestic industry to foreign trade, and (b) to define the advantage of “society” as that of one’s own nation. Is Smith not an advocate of free trade?

  1. Read all the way through this question before beginning your answers.
    1. In explaining the advance of knowledge in a science, one must choose between:
      1. The Kuhnsian view of revolutions, which says that wholly new paradigms (incommensurable with earlier paradigms) work major revolutions such as that of Marginal Utility, and
      2. All science is basically cumulative (which Kuhn believes is true only of “normal” science within a paradigm).
        Appraise these alternatives.
    2. Again, in explaining progress in a science one must choose between:
      1. A “great man” theory, in which a genius (he’s one by definition) makes a fundamental contribution and lesser scholars fill in the details, and
      2. The science has a main direction that is the product of the whole community of scholars. If a theory needs to be invented or discovered, one or more scholars will do so (Robert Merton).
        Again, appraise these alternatives.
    3. In both parts above, try to illustrate your argument by an episode in economics—preferably from this century. Thus, the theory of the firm, statistical study of economic functions, oligopoly theory, Keynes’ General Theory, monetarism, etc., are examples.

 

Source: University of Chicago Archives. George Stigler Papers, Addenda. Box 33 (2005-16), Folder “Misc. Course Materials. History of Economic Th[ought].”

Image Source: Posted by Glory M. Liu on her personal research webpage (next to the abstract for her article “Rethinking the Chicago Smith Problem: Adam Smith and the Chicago School, 1929-1980” published in Modern Intellectual History.

Categories
Chicago Exam Questions Fields History of Economics

Chicago. History of Economic Thought Ph.D Field Exam. Summer, 1974

 

The following examination consisting of six questions (answer five) comes from George Stigler’s papers at the University of Chicago Archives. It is safe to assume that Stigler penned these questions. 

The questions from the 1989 prelim on the History of Economic Thought are found in the following post.

________________________

History of Economic Thought
Summer, 1974

WRITE IN BLACK INK

WRITE THE FOLLOWING INFORMATION ON THE FIRST PAGE OF YOUR EXAMINATION PAPER:

— Your code number and not your name
— Name of examination
— Date of examination

Write only on one side of each page.

Write the following information on each following page of your examination paper:

— Top left: code number
— Top right: number of page

When you fold your paper at the end of the exam, write your code number on the back of the last page, and indicate total number of pages.

Results of the examination will be sent to you by letter.

 

Write on five of the following questions.

  1. John Stuart Mill is undergoing a rehabilitation of reputation after long being viewed as a pallid synthesizer of classical doctrines. Is this improved reputation deserved?
  2. Precisely how is the product-after-deduction-of-rent divided between labor and capital in the Ricardian system? Is the short run division different from the long run division? Is the system in equilibrium?
  3. Jevons is the founder of quantitative economics. What is the basis for this claim? Why did this type of work appear as late (or early?) as the 1860’s?
  4. Smith rated some forms of investment as socially preferable to others. What was his ranking of agriculture, manufactures and trade? Was his analysis valid?
  5. John A. Hobson, N. Lennin [sic], and others have authored theories of imperialism, which, in spite of various differences, have in common the proposition that modern expansionist wars and diplomatic entanglements are a consequence of the economic structure and dynamics of capitalism. Against this point of view, it has been argued that aggressive expansionism is much older than modern capitalism, and that economic interests have been used as a pawn of the political ambitions of statesmen. What kind of evidence would you regard as valid to evaluate the appropriateness of either type of theory on the relationships between economic change and war.
  6. Malthus’ gloomy prediction that the standard of living could not rise above a subsistence level proved wrong with respect to the Western world. List as many reasons as you can to account for this. Also, state as precisely as possible how Western population trends of the past two centuries can be related to (a) the law of diminishing returns and (b) shifts in production-possibility frontiers.

 

Source:  University of Chicago Archives. George Stigler Papers, Addenda. Box 33 (2005-16), Folder: “Exams & Prelim Questions.”

Image Source:  George Stigler page at the University of Chicago Booth School of Business website.

Categories
Chicago Exam Questions

Chicago. Price Theory. Ph.D. Core Examination. Summer, 1975

 

Graduate prelimary examinations for price theory at the University of Chicago for 1964 and 1969 have been transcribed and posted earlier. Economics in the Rear-view Mirror now adds the Summer quarter, 1975 exam to its stock of transcribed Chicago examinations.

__________________________________

Ph.D. Core Examination
PRICE THEORY
Summer, 1975

*  *  *  *  *  *  *  *  *

INSTRUCTIONS:

Write in black ink and write only on one side of each page.

Write the following information on the first page of your exam paper:

      • Name of examination
      • Date of examination
      • Your code number and not your name

Write the following information on each page of your examination paper:

      • Upper left: code number
      • Upper right: number of page

When you fold your paper at the end of the exam, write your code number on the back of the last page, and indicate total number of pages.

Results of the examination will be sent to you by letter.

ANSWER ALL QUESTIONS. TIME: 3 HOURS

*  *  *  *  *  *  *  *  *

  1. (60 minutes, 5 per item) Indicate whether each of the following statements is TRUE, FALSE, or UNCERTAIN. In each case write a few sentences explaining your answer. Your grade will be determined by your explanation.
    1. It is immediately obvious that if the firm has any significant degree of monopoly power, sales maximization would be better for the rest of the economy than profit maximization.
    2. When a firm increases its price because its raw material costs have risen, the buyers accept the price increase more readily.
    3. If A and B are produced in fixed proportions and consumed in fixed proportions, one of the two will be free.
    4. Marshall asserts that the rents of different qualities of agricultural land will approach equality as the economy grows in population and wealth.
    5. An industry whose output is increasing cannot be making negative profits.
    6. The prohibition on environmental pollution by (say) a factory cannot increase national income.
    7. A competitive industry is more likely to cartelize when the probability of expropriation increases.
    8. Regulation of a competitive industry by the government will decrease the probability of cartelization.
    9. In the social security systems of most countries, the age of retirement after which old age pensions are “payable” is lower for women than for men (usually 60 as compared with 65 years of age), even though on the average women live significantly longer than men. This is a clear case of discrimination against men, which should be protested by the Men’s Liberation Movement.
    10. The U.S. personal income tax system allows married couples to “split” their aggregate income equally and pay tax on the results at the same rates as single people would. This is a clear case of discrimination in favor of heterosexuality that should be vigorously protested by the Gay Liberation front.
    11. If the elasticity of supply is less than unity, and the elasticity of substitution in production greater than unity, a fall in the price of a factor must increase the demand for it.
    12. Labor can be “Exploited” only if there is monopoly in the product market.
  2. (25 minutes)
    In most states it is illegal for drug stores to advertise the prices of prescription drugs. A customer can find out the price of a prescription drug only by asking the pharmacist in person. In addition only pharmacists licensed by the state are allowed to dispense drugs and every drug store must employ at least one licensed pharmacist. One can become a licensed pharmacist by passing an examination administered by the state and written by a board of pharmacists. Finally, a pharmacist must fill a prescription exactly as it is written by the physician and may not substitute a generically equivalent drug.

    1. What would happen if pharmacists were allowed to advertise the prices of prescription drugs?
    2. What would happen to the price of drugs if pharmacists were allowed to substitute any drug from a specified list in place of the prescribed drug?
  1. (25 minutes)
    We are presently importing considerable oil at the $10 barrel price, and producing domestically at a free price from new wells and a $5 price from “old” wells (on amounts they produced before the oil price rises).

    1. What would be the effect on domestic price of a higher tariff on imports? On what would the magnitude of the price rise depend?
    2. What would be the effect on domestic price of a removal of the price ceiling on “old” oil? On what would the magnitude of this price effect depend?
  2. (25 minutes)
    Translate into the apparatus of indifference curves and budget lines the following phenomena:

    1. The individual likes good music more, the more he hears.
    2. The individual has monopsonistic power with respect to one commodity.
    3. (a) The consumption of the two commodities (however spaced) is poisonous.
      (b) The consumption of either commodity alone is poisonous.
    4. The individual cannot afford one of the commodities.
    5. (a) One of the commodities yields increasing marginal utility.
      (b) Both do.
  3. (20 minutes)
    1. Assume there is an exhaustible resource that can be extracted at a constant marginal cost c. Assume there is a competitive industry that extracts this resource. Derive the behavior of the equilibrium price over time if the demand schedule for the product remains constant over time.
    2. Under the same demand and cost conditions, derive the equilibrium price if the resource is controlled by a single firm.
  4. (25 minutes)
    Ontario imposes a tax of 30 percent on the sale or bequest of any land to non-Canadians. What are the effects of such a tax on:

    1. Landowners, Canadian and non-Canadian;
    2. Non-landowners, Canadian and non-Canadian.
      What will the effect be if leases are not regulated?

Source:  University of Chicago Archives. George Stigler Papers, Addenda. Box 33 (2005-16), Folder: “Exams & Prelim Questions.”

Image Source: The Quarter-Centennial Celebration of the University of Chicago (1916).

Categories
Chicago Economics Programs Northwestern

Northwestern. Robert J. Gordon selling Graduate Economics Program, 1977

 

The following soft-smile-hard-sell advertisement directed to potential applicants for the graduate economics program at Northwestern University was found in the economics department records of M.I.T. These notes were written by M.I.T. Ph.D. (1967) Robert J. Gordon who was then serving as the director of graduate admissions in economics for Northwestern in 1977-78. Gordon had been appointed professor of economics at Northwestern in 1973. This document provides a fascinating comparative glimpse of economics programs and locations as seen at that time.

Pro-tip:  Robert J. Gordon has shared his personal archive of “Photos of Economists”  on-line.

___________________

INFORMAL NOTES ON GRADUATE ECONOMICS AT NORTHWESTERN

Robert J. Gordon
Director of Graduate Admissions in Economics, 1977-78

To supplement the rather formal compilation of admissions and degree requirements contained in the pamphlet “Graduate Program in Economics, 1977-78”, I have put together a more informal set of observations on economics and graduate student life at Northwestern. Just as television commercials are no longer inhibited in naming competitors, I have included a few comparisons between Northwestern and some of the other departments to which you may be planning to apply. My comments on other departments are entirely personal based on my years as a student or faculty member at Harvard, M.I.T., and Chicago, and on the academic “grapevine” as regards other schools. In no sense do these subjective comparisons represent an “official view” of anyone in the administration of the Department of Economics or Northwestern University, nor would my colleagues necessarily agree with them.

I. WHO SHOULD APPLY TO NORTHWESTERN?

Universities in the United States are currently awarding roughly 2,000 M.A.’s and about 850 Ph.D.’s in Economics every year. Taking account of voluntary and involuntary dropouts, this suggests that about 2,500-3,000 candidates enter graduate school in Economics every year, and that the number of applicants is even larger. Both the applicants and the graduate schools are diverse in quality, and the admissions process can be described as an exercise in “matching” wherein the best schools attempt to select the best candidates, the next-best schools attempt to find the next-best candidates, and so on down the line. As long as a fee is charged for an admission application (currently $25 at Northwestern), potential applicants must carry out what economists call a “cost-benefit” analysis when deciding how many and which schools should be applied to. Too many applications may waste fees, but too few applications may lead to unanimous rejections. The best strategy is to realize that admissions committees are imperfect judges of your own “true quality”, and in some cases you also may over- or underrate yourself. To protect yourself against mistakes, it is best to apply not only to schools at your own perceived quality level, but also somewhat above and below. (In a recent survey 57 percent of the respondents rated themselves in the top ten percent of their class!)

Rating Department “Quality”

While a number of different characteristics are relevant to the final choice, faculty quality is the most important single criterion by which alternative graduate programs should be judged. Among the advantages provided by faculty members who are widely regarded in the profession as among the best in their field are not only correct and current courses, but also guidance in Ph.D. dissertations and knowledge of the most promising areas for student research, the ability to win research grants which in most cases provide funds for student research assistantships, and finally, widespread professional contacts to aid students in the job market.1

_______________

1By the way, there is currently no problem in finding jobs after graduate school in Economics. This contrasts with other disciplines, particularly the humanities, history, and sociology, where jobs are scarce and some Ph.D.’s are unemployed. The healthier job market in Economics is explained by the large demand for Ph.D. economists in business and government which supplements the demand by colleges for teaching posts.

_______________

The already high quality of the Northwestern faculty has been supplemented in the last few years by the arrival of three new full professors who are both relatively young and are regarded as among the top economists in their respective fields—Marc Nerlove (winner in 1969 of the American Economic Association’s John Bates Clark award for the best American economist under the age of 40), Frederick Scherer, and myself. Since the most recent official survey to determine the ranking of Northwestern relative to other economics departments is more than five years out of date, there is no accurate information available which is both objective and current.

As a substitute I can provide the results of my own subjective but detailed evaluation, which is current as of Fall 1976 (e.g., it takes account of the movements of J. Stiglitz from Stanford to Oxford and Michael Rothschild from Princeton to Wisconsin). In consultation with several highly regarded economists, all permanent faculty members in the top 18 U.S. departments have been rated with a “quality score” ranging from one (low) to 10 (superman), and the total scores in each department of the faculty members rating “5” or above have been added up.2 An attempt has been made to include members of business schools known to play a major role in graduate economics education. For most departments official faculty lists have been obtained to insure completeness.

_______________

2Note that this technique gives a premium to large departments, partially explaining the “victory” of Harvard over M.I.T.

_______________

Department

Rating Points

Citations3

1.

Chicago 152 (1574)
2. Harvard 147

(1472)

3.

M.I.T. 139 (1241)
4. Yale 122

(598)

5.

Northwestern 97 (401)
6. Princeton 96

(362)

7.

Pennsylvania 93 (509)
8. Wisconsin 85

(587)

9.

Berkeley 75 (420)
9. Stanford 75

(402)

11.

Minnesota 72 (209)
12. U.C.L.A. 70

(344)

13.

Rochester 43
14. Columbia 41

(454)

14.

Maryland 41 (276)
16. Michigan 40

17.

Carnegie-Mellon 38
18. Brown 23

_______________

3Numbers in parentheses are faculty citations in the 1973-74 Social Sciences Citation Index.

_______________

It would appear that there are four departments in the top category, and then a group of “next best” from ranks 5 through 12 which are very close together in total points. If you think rather highly of yourself, it is probably worthwhile to apply to at least one department in the “top four,” but keep in mind that the total number of first-year students in these departments is only about 135 out of the 2,500-3,000 students who enter graduate school in economics each year. Most students will want to apply to one or more of the “next best,” whether they are top students who want a safety valve, or whether they evaluate themselves at “next best,” or whether they think of themselves as “third best” but are willing to take a chance that an admissions committee in the second tier might overrate them.

Comparisons Among Departments

Selection of a choice among the “top four” depends on your abilities and tastes. M.I.T. is almost universally praised for the quality of its faculty, its devotion to the teaching of graduate students, and for its physical facilities, but it can accept only about 35 out of roughly 350 applications, and students without excellent mathematical training will feel left behind. Harvard has a senior faculty which is tops in fame and reputation but which is frequently criticized as aloof and inaccessible not only to students but even to junior faculty members; classrooms and faculty offices are in several buildings with no natural physical focal point for students; but on the other hand the attractions and convenience of Cambridge have appeal. At Chicago the faculty is better at teaching than at Harvard, is much more accessible, and in many fields of economics is more innovative than at M.I.T.; compared to M.I.T. Chicago’s disadvantages are huge first-year classes (55-80 is typical) and the neighborhood (crime is a problem, and also there is much less to do in Hyde Park as compared with Cambridge, so one is dependent on downtown Chicago, which is very difficult to reach by public transportation from Hyde Park at night). I lack personal experience at Yale—the problems which recur in “grapevine” conversations is the physical and social separation between the faculty in the Cowles foundation and in the rest of the departments, the aloofness of many faculty members, and the disadvantages of living in New Haven. On the other hand, some ex-graduate students claim that the Department/Cowles split does not affect them, even if it has disadvantages for faculty members.

How does Northwestern compare with its competitors in the “next best” group? Stanford, Berkeley, and U.C.L.A. are obviously superior in climate but suffer from other disadvantages. Because the Stanford campus is so vast, there is no university shopping district within easy walking distance, and the attractions of Palo Alto are uninterestingly suburban, with the delights of San Francisco 35 miles away and accessible only by car. Berkeley is a much better place to live, both more interesting by itself and closer to San Francisco, but the department itself is large and impersonal, with long corridors of closed office doors, and the mathematical economists are off across the campus in a separate building. Princeton is located in a posh expensive small town 45 miles from New York, which is therefore less accessible than Boston from Cambridge, San Francisco from Berkeley, or Chicago from Evanston. Pennsylvania is located in a relatively unattractive section of Philadelphia and faculty houses are widely dispersed (as is true at Harvard and M.I.T.), which inhibits the faculty from lingering after seminars and from giving post-seminar cocktail parties. I lack close familiarity with U.C.L.A., Penn, and Minnesota and won’t cast further aspersions, other than to note that they are all relatively large and impersonal universities.

Northwestern combines a number of advantages—a high-quality faculty which is extremely accessible to students both individually and in group seminars, together with a location which combines the best features of small-town and large-city living. Another significant strength is the relatively prosperous budgetary situation at Northwestern, which is currently allowing the Economics Department to embark on a major program of hiring new tenure and nontenure faulty members. It is likely that by the time current applicants arrive here, the relative ranking of Northwestern’s Economics Department will have risen even higher relative to the many universities which are currently suffering from tight budgets.

 

 II. ASPECTS OF GRADUATE LIFE AT NORTHWESTERN

Courses and Seminars

The Ph.D. program typically takes four years, divided into an initial two-year period devoted mainly to courses, followed by two additional years devoted to attending seminars, finding a thesis topic, and writing the dissertation. There is a single written general examination (“prelim”) in economic theory (three hours for macro and three hours for micro), which most students take after their first year of courses. The process of learning at Northwestern does not consist of rote learning or indoctrination, but rather a process by which the student is first trained in the tools of theoretical, mathematical, and statistical analysis, and then is exposed to the frontiers of economic science and urged to use his tools to help resolve controversies and contribute to the advance of knowledge.

The process by which a student arrives at a dissertation topic generally begins in the second year of class work. Unlike many graduate schools, where the general exam process continues to the end of the second year and sometimes beyond, at Northwestern most students enter the second year of classes with their general exam behind them and can concentrate on finding a special field of interest. Second year classes are usually small enough to allow students to participate actively in discussion and to encourage the faculty both to assign term papers and to read them carefully. Second-year term papers are a “proving ground” where students can experiment with possible thesis topics. Under a new system, students are required to give a paper in a field “workshop” after they are finished taking the courses their primary field sequence.

A formal requirement for admission to Ph.D. candidacy, in addition to the written general examination on economic theory, is an oral qualifying examination on the dissertation, which is usually taken during the third year. The purpose of the exam is to ascertain whether the dissertation topic chosen by the student is feasible. Usually the exam consists of a discussion of a brief written thesis proposal which a candidate submits to the faculty committee of examiners.

At many graduate schools there is no formal program for third and fourth year students, who simply “disappear” in the library or their homes and are unavailable for conversation and consultation with each other. At Northwestern, on the other hand, there is an active workshop program to provide forums where graduate students are exposed to new ideas and have a chance to see each other regularly. A centerpiece is the Tuesday night student- faculty seminar, where students present both early and finish versions of their dissertation research, with a substantial cross-section of the faculty attending regularly to provide advice and criticism. Research seminars in macroeconomics and labor, in applied micro economics, industrial organization and in mathematical economics (in collaboration with faculty members at the Graduate School of Management) meet regularly for presentation and discussion of papers by faculty, students, and visitors. These seminars are not only a major channel of communication between faculty and students, but are also an important method of intellectual interchange among faculty members. In addition, there is a regular visiting speakers program, in which well-known faculty members from other universities are invited to Northwestern to present talks on their research Evanston’s location also facilitates additional informal seminars by visitors who are traveling through the Chicago area.

Particular Strengths

Although the Economics Department teaches graduate courses in all of the major fields of economics, it has particular areas of strength in which faculty members are currently making a major research contribution:

Microeconomic Theory
Mathematical Economics
Econometrics
Theoretical and Applied Macroeconomics
Labor Economics
Public Finance
Managerial Economics and Industrial Organization
Transportation and Urban Economics
Economic History
Medical Economics
Economics of Population and the Family

Faculty and Student Accessibility

Faculty-student contact is unusually good at Northwestern for a number of reasons. Although the classes in economics theory in the first quarter are fairly large, since some management school students are required to attend, for the remaining two quarters of the first year the theory classes typically contain only 25. Second-year class enrollments are often in the range of 5 to 10, allowing a workshop atmosphere and considerable faculty attention to the individual student term papers and research projects. Most of the faculty live close to the Evanston campus and typically hold open cocktail parties in their homes after seminars by visiting speakers. After the Tuesday night faculty-student seminar, both students and faculty regularly adjourn to a local pub (this never happens at M.I.T., Harvard, or at other departments where faculty residences are located in distant suburbs). Many third-and fourth-year students have offices adjoining faculty offices and see their faculty neighbors regularly during coffee breaks. Another advantage promoting easy interaction is the relative youth of many of the tenured faculty, in contrast to the older “stars” at some other departments who spend more time consulting in Washington than talking to their students.

Easy contact among students is even more important than faculty-student contact in the first year, when students need to get to know each other and form into small study groups. This is facilitated at Northwestern by a graduate student lounge in the basement of 1922 Sheridan Rd. (the main economics building), where coffee is available and students are encouraged to study or talk between classes. Another convenience is the Library, completed in 1970 and about a 3-minute walk from the main department building. A special feature of the uniquely designed library is the divisional arrangement of books and journals in three research towers, one for the social sciences. On each of the circular levels of the research towers, ranges of books in specialized journals are placed in a radial pattern. At the periphery of each circle surrounding the collections is a repeating series of carrels, typing rooms, graduate and faculty studies, and seminar rooms in close proximity to the main body of printed materials needed by the various disciplines. (The computer center is also a great advantage, as it is relatively well-run and provides fast 15-20 minute “turnaround time” except in peak weeks at the end of the quarter).

Faculty and Courses in the Graduate School of Management

Although other universities also have business schools, of course, Northwestern’s provides a particular asset because of its unusual orientation toward economics and because of the unusually close contact between members of the Economics Department and the Management School. Economics topics covered in Management School courses include optimization theory and techniques, decision-making under uncertainty, models of production and technology, models of financial decision-making, and others. Management School courses are open to economics students, and dissertation committees often include Management School members.

Research Centers

Several “centers” headed by Department faculty members support and encourage research in their areas, provide offices and secretarial help, and arrange seminars by resident faculty, students, and visitors. Research Centers have been established in mathematical economics, transportation economics, and urban affairs.

 

III. ADMISSIONS AND FELLOWSHIPS

The formal admissions procedure is described in the “Graduate Program” pamphlet. Prospective applicants should note that they are required to submit scores from the Graduate Record Examination only for the verbal and quantitative aptitude tests and not for the advanced test in economics. This is consistent with our desire to encourage applications from those who have not chosen to major in economics as undergraduates. Our interest is in finding motivated, intelligent students with enough quantitative aptitude to understand economics theory and enough curiosity about the world around them to do creative economic research.

All available criteria are used by the Admissions Committee (myself and a few colleagues) to evaluate each applicant — undergraduate grade record, letters of recommendation, the applicant’s score on the Graduate Record Examination, and special factors. No arbitrary boundaries are established for grades or GRE scores. Applicant should encourage those writing letters of recommendation to be as specific as possible, a process which can be facilitated if applicants confer with the letter writers regarding their strengths and weaknesses. Applicants who have any unique qualifications or wish to explain “soft spots” in their grade record are encouraged to file supplementary statements with their applications.

Since first-year calculus is essential and second-year calculus is extremely useful for the study of economics, prospective applicants who have not yet taken these courses for credit are urged to do so at some time between now and their arrival at graduate school (wherever they choose to attend). The summer before arrival is an excellent time to take an extra course, and second-year calculus should receive top priority.

The selection of fellowship winners—for both university and department fellowships—is made by the Admissions Committee shortly after the admissions decision. A number of fellowships are also available under a Rockefeller-sponsored Northwestern Program in the Economics of Population and Household Behavior. To maximize their chances of receiving support, applicants are urged to apply for several of the fellowships awarded by outside foundations, businesses, and government agencies. Do not despair if you do not receive the fellowship, for there are several other alternatives. Most obvious is the student loan program, through which students can borrow money to cover most or all of their tuition (for details, see the Northwestern Graduate School Catalog). A substantial portion of the loan funds is available at three percent interest, which in these days of inflation represents a negative “real” rate of interest. Remember also that inflation reduces the real value of the principal to be repaid. Other sources are part-time academic year jobs, research assistantships for faculty members (usually reserved for students in the third and fourth year), and support from parents and/or spouse (now that women have been liberated, the Ph.T. Degree—“putting hubby through”—has been supplemented by the Pw.T,—“putting wife through”). We do not at present normally award teaching assistantships to first-year students.

 

IV. LIVING IN EVANSTON AND CHICAGO

Evanston

Evanston is the first suburb north of Chicago along Lake Michigan, and the Evanston-Chicago boundary is located 9.5 miles north of the Chicago “Loop.” Despite its proximity to Chicago, Evanston’s aesthetic attractions are immediately apparent when one crosses north over the city line. All of it streets are lined with unusually grand old shade trees; street lights are old-fashioned; the downtown shopping area is free of overhanging neon signs and decorated by city-maintained flowerbeds; and the lakefront is lined with the bicycle path, parks, and beaches where swimming is safe in unpolluted water.

With a population of about 80,000, Evanston is about the same size as Berkeley and Cambridge and shares their advantage of combining the convenience and call of a relatively small self-contained city with the entertainment and cultural attractions of a large urban center. Its residents include not only students and professors, but also sizable numbers of lawyers, architects, and other professionals who help to support groups and organizations in music, politics, and other areas. Student housing is available both in private and university-owned buildings (see the Graduate School Catalog for details), and most students are able to live within a short walk or bicycle ride from Northwestern’s lakefront campus.

Evanston’s downtown shopping area begins immediately south of the campus, with a group of books stores located across the street from the main university administration building. Shopping opportunities are unusually diverse for a city of Evanston’s size, with several branches of downtown department and specialty stores, large supermarkets and small “gourmet” food shops, and a variety of shops selling both standard and esoteric clothes, furnishings, and other items.

Transportation within Evanston is easy whether or not students own cars. Parking is available on side streets and in public parking garages downtown. Since most side streets are relatively free of traffic, many students prefer to rely entirely on bicycles for travel within Evanston. The public transit fare is subsidized by the City Council at 25¢ for travel within Evanston on four bus lines and on the rapid transit stations which shuttle at five-minute intervals along a north-south axis which skirts the western boundary of the campus and continue south to the Chicago border and on to downtown Chicago  (see below).

Many Evanston residents formerly lived in the Hyde Park and South Shore districts of Chicago—adjacent to the University of Chicago—but moved north to escape the South Side crime problem. Evanston is fortunate in its low crime rate, less than half the rates recorded in Berkeley and Cambridge in the 1970-73 period, and is a place where both students and faculty feel perfectly free to walk out at night. The only noticeable disadvantage of life in Evanston is the climate between November 15 and March 15, when the average daily high-temperature is about 35 degrees (i.e., five degrees colder than New York). Average annual snowfall is a bit more than in New York and a bit less than in Boston. The weather during the rest of the year similar to that in the northeastern quadrant of the U.S. Over all the weather is obviously no match for Berkeley, about the same as Boston and decidedly better than Madison or Minneapolis.

Chicago

From the Northwestern campus the center of Chicago is 25 minutes by car via Lakeshore Drive, and is almost easily accessible via rapid transit trains which stop twice at the western edge of the campus and reach the “Loop” in 30 minutes during rush hours, and about 40 minutes at other times. These times overstate the duration of travel to many restaurants, theaters, and clubs, the majority of which are located on the North Side of the city, i.e., between the “Loop” in the Evanston border. Trains run all night, and at most hours their frequency is every five to ten minutes.

Until six years ago I had never been to Chicago and had an irrational fear of the unknown Midwest, which may be shared by some prospective applicants from the East and West Coasts. My years of sampling Chicago’s attractions have converted me, and perhaps you will be interested in some personal opinions and comparisons:

    • The main aesthetic attractions are (1) the Loop and Near North Side, containing some of the best urban architecture in the world, in (2) the 20-mile bicycle path along the lake front, which is a continuous band of parks, beaches, and yacht harbors.
    • The major museums are all very large and among the top two or three in the country, including the Art Institute, the Field Museum of Natural History, the Planetarium, the Aquarium, and the Museum of Science and Industry, the latter having special appeal for any economist interested in the history of technical change and in “how things work.” There are smaller art galleries as well, and a local “school” of modern art, which I saw exhibited in Mexico City as “La Nueva Escuela de Chicago.”
    • In New York visiting concerts of Georg Solti with the Chicago Symphony have become, according to the New York Times, the “most eagerly anticipated musical events since Toscanini.” The Symphony plays three concerts a week during the academic year at Orchestra Hall in the “Loop” and frequent concerts during the summer at the Ravinia Festival in a suburb a few miles north of Evanston. The Lyric Opera presents a three-month season in the fall and shares with San Francisco the top rank among US opera companies outside of New York. There are several local chamber music groups and a long list of touring concert artists, including the major New York ballet companies, which perform throughout the year. There are also three full-time FM classical music stations.
    • The “club scene,” both night clubs and coffee houses, is unsurpassed among cities outside of New York, and the blues and folk music offerings surpass New York. Each Friday a free newspaper, the Reader, lists about 150 blues, folk, rock, and jazz acts appearing in local clubs.
    • Speaking of newspapers, the Chicago Tribune has dropped its conservativism of the Col. McCormick days and was recently named one of the country’s 10 best by Time. Its local news and features are excellent, although it still can’t compare for national and international news with the New York Times (the latter is flown in daily for purchase at Evanston newsstands or for home delivery).
    • In the restaurant category Chicago ranks after New York, San Francisco, and perhaps New Orleans. Its best are not as good as in those three cities, but that doesn’t matter much for students who can’t very often afford $50 French dinners. More important and interesting is Chicago’s strength, the hundreds of inexpensive “storefront ethnic” restaurants, many of which are in the north part of Chicago close to Evanston. Take your choice among German, French Provincial, East European Italian, Cantonese, Mandarin, Korean, Thai, Japanese, Indian, Mexican, Cuban, and Peruvian.
    • Cheap entertainment is available at the student-run film societies at Northwestern, other colleges, and the Art Institute, and at a number of commercial theaters on the north side of Chicago which only charge $.75 or $1.00 for a double bill of second- or third-run features.
    • While the quantity of live drama is no match for New York, there is a surprisingly broad offering by two accomplished professional repertory groups, a number of “off-Broadway” and experimental groups, and both pre- and post-Broadway touring shows. In recent years a number of shows have “graduated” to New York after starting here.

****

Obviously the preceding notes will only begin to answer your questions. Write me for any additional information you need. I’ll respond without delay if I know the answer or else I’ll find a colleague who can advise you. I can also arrange for a current graduate student to provide more information on student reactions.

 

Source: M.I.T. Archives. Records of the Department of Economics. Box 3, Folder “Quality Rating.”

Image Source: Robert J. Gordon at First Bank of Japan Monetary Conference, June 1983. Detail from picture with James Tobin.

 

 

 

 

 

 

 

 

 

 

 

Categories
Exam Questions Johns Hopkins M.I.T. Suggested Reading Syllabus

M.I.T. Readings and exam questions for fiscal and monetary policy. Domar, 1957

Evsey Domar’s first semester at M.I.T. was as a visiting professor according to the teaching records of the economics department. He taught one seminar on Russian Economics (14.292) and a graduate course with the nominal title “Fiscal Policy”. That course had been taught previously by E. Cary Brown (Spring 1954, 1955) and R. A. Musgrave, visiting Professor (Spring 1956).

Inspection of the ten-page course bibliography and the final examination questions along with two note-cards filed with these course materials, it appears that well over half the course was in all likelihood dedicated to fiscal policy topics with monetary policy for stabilization topics accouting for perhaps one-third of the course. Just as the length of the course bibliography (typical for Domar) is daunting, his use of asterisks to designate recommended reading was exceedingly liberal. An examination of the final examination questions leads me to conclude that it should be rather easy to reduce the course reading list (for examination purposes!) to less than two pages.

___________________________

Course Enrollment
(Second term, 1956-57

Instructor

Domar, E. D.

Rank

Prof. (Visit.)

Subj. No.

14.472

Subj. Title

Fiscal Policy

No. Class Hours/Week

3

No. Students

22

Source: M.I.T. Archives, Department of Economics Records 1947-, Box 3, Folder “Teaching Responsibility”.

___________________________

Typed notecards for an introduction to or a review of course.

The traditional arguments regarding the purposes of Monetary Policy:

  1. Stabilization of general prices or of factor earnings—the Wicksell-Davidson controversy. The instrument was the relation between the natural and the market rates.
  2. Stabilization of prices or of employment. Recent literature is full of this.
  3. Stabilization of the general prices or of prices of Federal securities. See Douglas’s and other reports on this recent controversy.
  4. Stabilization of employment or the achievement of growth. Any conflict?
  5. Discretionary methods or automatic provisions? See Simons’ article in Readings in Monetary Theory.
  6. To provide credit and currency, sound and in sufficient quantity.
  7. To protect the international position of the country.
  8. To have special effects, such as:

a. by region
b. by industry
c. by commodity consumed (such as tobacco) or housing
d. on population (by giving exemptions or subsidies for dependents)

  1. Provide revenue [handwritten addition]
  2. Distribution of income [handwritten addition]

*  *  *  *  *  *  *  *  *  *  *  *  *

The following limitations, some real, other imaginary, explain why Fiscal Policy is not as simple as Lerner makes it:

Income distribution
Size of the deficit
Size of the budget
Balance of payments
Special regional and industrial effects
Effects on incentives to work (in inflation)
Automaticity of the system (built-in-flexibility)
Monetary effects (on reserves, deposits)
Long-run effects (on growth and development)

Their presence complicates things and explains all the ingenious articles and tax devices frequently suggested. If not for them, fiscal policy would be very simple indeed: cut taxes or increase taxes, and the same with expenditures.

___________________________

READING LIST
14.472 Fiscal Policy
Spring Term 1956-57

Professor E. D. Domar

PART I—MONETARY POLICY

The purpose of this list is to suggest to the student the sources in which the more important topics in Monetary Policy are discussed from many points of view. His objective should be the understanding of these topics and not the memorization of who said what.

Most of the sources listed here, and particularly the Congressional materials, discuss a number of questions not only in Monetary but in Fiscal Policy as well. Hence it is difficult to classify them.

Items marked with an * are strongly recommended. (I don’t like to use the expression “required” in a graduate reading list.)

  1. Factual Materials on Monetary Problems

Federal Reserve Bulletin.

Treasury Bulletin.

Annual Reports of the Secretary of the Treasury and of the Board of Governors of the Federal Reserve System.

Historical Statistics of the United States, 1789-1945, and the Continuation to 1952.

Congressional Hearings, Reports and other Materials listed below.

  1. Introduction

Hart, Albert Gailord, Money, Debt and Economic Activity, New York 1948.

Hicks*, J. R., “A Suggestion for Simplifying the Theory of Money,” Economica, 1935; reprinted in Readings in Monetary Theory.

Lerner*; Abba P., “Functional Finance and the Federal Debt,” Social Research, 1943, and Readings in Fiscal Policy, p. 468, also Chapter 24 in his Economics of Control, New York, 1944.

Poole, Kenyon E., ed. Fiscal Policies and the American Economy.

Sproul* Allan, “Changing Concepts of Central Banking,” Money, Trade and Economic Growth in Honor of John Henry Williams, New York, 1951.

  1. Monetary Theory and Growth

Gurley*, John G. and Shaw, E. S., “Financial Aspects of Economic Development,” American Economic Review, September 1955, pp. 515-538.

  1. Effectiveness of the Interest Rate

Ebersole*, J. F., “The Influence of Interest Rates,” Harvard Business Review, XVII, i, 1938, pp. 35-39.

Henderson*, R. D., “The Significance of the Rate of Interest,” Oxford Economic Papers, October 1938, I, pp. 1-13.

*Meade, J. E. and Andrews, P. W. S., “Summary of Replies to Questions on Effects of Interest Rates,” Oxford Economic Papers, October 1938, I, pp. 14-31.

Sayers, R. S. “Business Men and the Terms of Borrowing,” Oxford Economic Papers, Feb. 1940, III, pp. 23-31.

Andrews, P. W. S., “A Further Inquiry into the Effects of Rates of Interest,” Oxford Economic Papers, Feb. 1940, III, pp. 32-73.

White*, William H., “Interest Inelasticity of Investment Demand—The Case from Business Attitude Survey Re-Examined,” American Economic Review, September 1956, pp. 565-87.

Lutz, Friedrich A., “The Interest Rate and Investment in a Dynamic Economy,” American Economic Review, Dec. 1945.

  1. General Surveys of Monetary Policy

Federal Reserve Board*, Tenth Annual Report for 1923. See pp. 29-39 particularly.

Chandler*, Lester V., “Federal Reserve Policy and the Federal Debt,” American Economic Review, 1949, and Readings in Monetary Theory, p. 394.

Hardy, Charles O., “Fiscal Operations as Instruments of Economic Stabilization,” American Economic Review, Supplement, 1948, pp. 395-403 and Readings in Monetary Theory, p. 394.

Hart, Albert Gailord, “Monetary Policy for Income Stabilization,” Income Stabilization for a Developing Democracy, ed. by Max F. Millikan, New Haven, 1953.

Williams, John H., “The Implications of Fiscal Policy for Monetary Policy and the Banking System,” AER Proceedings, March 1942; Readings in Fiscal Policy, p. 185.

Smith*, Warren L., “On the Effectiveness of Monetary Policy,” American Economic Review, September 1956, pp. 588-606.

  1. Suggested Objectives and Policies

Hammarskjold, Dag, “The Swedish Discussion on the Aims of Monetary Policy,” reprinted in International Monetary Papers, No. 5, pp. 145-154.

Simons*, Henry C., “Rule versus Authorities in Monetary Policy,” JPE, 1936, and Readings in Monetary Theory, p. 337.

Simons, Henry, “On Debt Policy,” JPE, Dec. 1944, and Readings in Fiscal Policy.

Mints*, Lloyd, W., “Monetary Policy,” Review of Econ. and Stat., 1946 and Readings in Fiscal Policy, p. 344.

Bach*, G. L., “Monetary-Fiscal Policy Reconsidered,” JPE, Oct. 1949, and Readings in Fiscal Policy.

Friedman*, Milton, “A Monetary and Fiscal Framework for Economic Stability,” American Economic Review, 1949, and Readings in Monetary Theory, p. 369.

*United Nations. National and International Measures for Full Employment. Report by a group of experts appointed by the Secretary-General (Lake Success, New York, December 1949).

Viner*, Jacob, “Full Employment at Whatever Cost,” QJE, August 1950, pp. 385-407. Reproduced with omissions in Economic Policy, Readings in Political Economy, edited by William D. Grampp and Emanuel T. Weiler, Homewood, Ill., 1956, pp. 54-65.

Samuelson* Paul A., “Principles and Rules in Modern Fiscal Policy: A New-Classical Reformulation,” Money, Trade and Economic Growth in Honor of John Henry Williams, New York, 1951.

Seltzer* Lawrence H., “Is a Rise in Interest Rates Desirable or Inevitable,” American Economic Review, Dec. 1945; Readings in Fiscal Policy, p. 202.

Roosa, Robert V., “Interest Rates and the Central Bank,” Money, Trade and Economic Growth in Honor of John Henry Williams, 1951.

Roosa*, Robert V., “Integrating Debt Management and Open Market Operations,” American Economic Review, 1952, and Readings in Fiscal Policy, p. 265.

Hansen*, Alvin H., “Monetary Policy,” The Review of Economics and Statistics, May 1955, pp. 110-119.

  1. Commodity Money

Graham, Benjamin, World Commodities and World Currency, New York 1944.

Graham*, Frank D., “Full Employment without Public Debt, Without Taxation, Without Public Works, and without Inflation,” Planning and Paying for Full Employment, edited by Abba P. Lerner and Frank D. Graham, 1946.

  1. Congressional Materials

Joint Committee on the Economic Report. Money, Credit, and Fiscal Policies. Hearings before the Subcommittee on Monetary, Credit and Fiscal Policies of the Joint Committee on the Economic Report, 81st Congress, First Session, September 23, November 16,17,18,22,23 and December 1,2,3,5,7, 1949.

Joint Committee on the Economic Report. Monetary, Credit, and Fiscal Policies. A Collection of Statements Submitted to the Subcommittee on Monetary, Credit and Fiscal Policies by Government Officials, Bankers, Economists, and Others. 1949.

Joint Committee on the Economic Report (The Douglas Subcommittee). A Compendium of Materials on Monetary, Credit, and Fiscal Policies. A Collection of Statements Submitted to the Subcommittee on Monetary, Credit, and Fiscal Policies by Government Officials, Bankers, Economists, and Others. 81st Congress, 2ndSession, Senate Document No. 132, 1950.

Joint Committee on the Economic Report*. Monetary, Credit, and Fiscal Policies. Report of the Subcommittee on Monetary, Credit, and Fiscal Policies of the Joint Committee on the Economic Report. 81st Congress, 2ndSession, Senate Document No. 129, 1950.

Joint Committee on the Economic Report. Monetary Policy and the Management of the Public Debt Hearings before the Subcommittee on General Credit Control and Debt Management of the Joint Committee on the Economic Report, 81st Congress, 2nd Session, March 1952.

Joint Committee on the Economic Report. Monetary Policy and the Management of the Public Debt. Their Role in Achieving Price Stability and High-Level Employment. Replies to questions and other material for the use of the subcommittee on general credit control and debt management. 82nd Congress, 2nd Session, Senate Document No. 123, 1952.

Joint Committee on the Economic Report. Monetary Policy and the Management of the Public Debt Report of the Subcommittee on General Credit Control and Debt Management of the Joint Committee on the Economic Report, 82nd Congress, 2nd Session, 1952.

Joint Committee on the Economic Report. United States Monetary Policy: Recent Thinking and Experience Hearings before the Subcommittee on Economic Stabilization of the Joint Committee on the Economic Report. 83rd Congress, 2nd Session, December 6 and 7, 1954.

Joint Committee on the Economic Report. January 1956 Economic Report of the President. Hearings before the Joint Committee on the Economic Report. 84th Congress, 2nd Session, January 31, February 1,2,3,6,7,8,9,14,15,17 and 28, 1956.

Joint Committee on the Economic Report*. Conflicting Official Views on Monetary Policy; April 1956. Hearings before the Subcommittee on Economic Stabilization of the Joint Committee on Economic Report, 84thCongress, 2nd Session, June 12, 1956.

  1. Readings for Amusement

Outside Readings in Economics, second edition. Selected by Hess, Arleigh P. Jr., Gallman, Robert E., Rice, John P., and Stern, Carl, New York, 1956. The “Dialogue on Money,” by D. H. Robertson; “The Island of Stone Money,” by William H. Furness III; “The Paper Money of Kubla Khan,” by Marco Polo; and “The Edict of Diocletian,” by Humphrey Mitchell, pp. 314-335 are very amusing and instructive.

 

PART II—FISCAL POLICY

See the remarks in Part I.

  1. Factual Materials of General Character

Joint Committee on the Economic Report.* The Federal Revenue System: Facts and Problems, 1956

Treasury Bulletin

Annual Reports of the Secretary of the Treasury and of the Commissioner of Internal Revenue

Statistical Abstract of the United States

Historical Statistics of the United States, 1789-1945 (published by the U. S. Bureau of the Census)

U. S. Treasury Department, Internal Revenue Service, Statistics of Income (an annual publication in two volumes)

U. S. Bureau of the Census, Summary of Governmental Finances (annual series)

The Budget of the U. S. Government

Commerce Clearing House, Inc., Tax Systems

West Publishing Co., Federal Tax Regulations, 1956

Congressional Hearings and Reports, listed in Part I and below

Textbooks on Public Finance and Fiscal Policy

  1. Historical Studies

Ratner, S., American Taxation: Its History as a Social Force in a Democracy, New York, 1942

Fabricant*, S., The Trend of Government Activity in the United States since 1900, New York, 1952, Chapters 1, 6, 7

Studenski, P. and H. E. Kroos, Financial History of the United States, New York, 1952

Musgrave*, R. A. and J. M. Culbertson, “The Growth of Public Expenditures in the United States,” National Tax Journal, June, 1953, pp. 97-115

Paul, R. E., Taxation in the United States, Boston, 1954

  1. Fundamental Assumptions

Hansen*, A. H., “The Stagnation Thesis,” Fiscal Policy and the Business Cycle, New York, 1941, pp. 38-46, and Readings in Fiscal Policy

Schumpeter*, J. A., “Economic Possibilities in the United States,” Capitalism, Socialism, and Democracy, 1947, and Readings in Fiscal Policy

Domar*, E. D., “The Problem of Capital Accumulation,” The American Economic Review, December, 1948

Fellner*, W., “Relative Emphasis in Tax Policy on Encouragement of Consumption or Investment,” Federal Tax Policy for Economic Growth and Stability, Joint Committee on the Economic Report, Washington, D.C., November 9, 1955, p. 210

Hansen*, A. H., “Economic Stability and Growth,” Federal Tax Policy for Economic Growth and Stability, Joint Committee on the Economic Report, Washington, D. C., November 9, 1955, p. 14

Smithies*, A., “Economic Growth as a Policy Objective,” Federal Tax Policy for Economic Growth and Stability, Joint Committee on the Economic Report, Washington, D. C., November 9, 1955, p. 32.

  1. General Objectives and Policies

Keynes* J. M., “An Open Letter,” The New York Times, 1933, and Readings in Fiscal Policy

Lerner*, A. P., “Functional Finance and the Federal Debt,” Social Research, 1943, and Readings in Fiscal Policy. Also Chapter 24 in his Economics of Control, New York, 1944

Hart, A. G., “’Model-Building’ and Fiscal Policy,” American Economic Review, 1945, and Readings in Fiscal Policy

Committee for Economic Development, “Taxes and the Budget: A Program for Prosperity in a Free Economy,” Readings in Fiscal Policy, 1947

Colm* G., “The Government Budget and the Nation’s Economic Budget,” Public Finance, 1948, and Readings

Friedman*, M., “A Monetary and Fiscal Framework for Economic Stability,” American Economic Review, 1948, and Readings

National Planning Association, “Federal Expenditure and Revenue Policy for Economic Stability,” 1949, Readings

Bach*, G. L., “Monetary-Fiscal Policy Reconsidered,” Journal of Political Economy, October, 1949, and Readings

United Nations*, National and International Measures for Full Employment (report by a group of experts appointed by the Secretary-General), Lake Success, New York, December, 1949

Simons*, H. C., Federal Tax Reform, Chicago, 1950

Viner*, J., “Full Employment at Whatever Cost,” The Quarterly Journal of Economics, August, 1950

Samuelson*, P. A., “Principles and Rules in Modern Fiscal Policy; A Neoclassical Reformulation,” Money, Trade, and Economic Growth: in Honor of John H. Williams, New York, 1951

Millikan, M., ed., Income Stabilization for a Developing Democracy, Yale, 1953

Rolph, E.R., The Theory of Fiscal Economics, Berkeley and Los Angeles, 1954

U. S. Congress, Joint Committee on the Economic Report, Federal Tax Policy for Economic Growth and Stability, December, 1955, Hearings

American Economic Association* Readings in Fiscal Policy, Homewood, Illinois, 1955

National Bureau of Economic Research, Policies to Combat Depression, a conference of the Universities-National Bureau Committee for Economic Research, 1956

Council of Economic Advisers*, the latest Annual Report

  1. Institutional Factors

Bailey, S. K., Congress Makes a Law: the Story behind the Employment Act of 1946, New York, 1950

Bailey, S. K. and H. D. Samuel, Congress at Work, New York, 1952

Blough, R., The Federal Taxing Process, New York, 1952

Smithies*, A., The Budgetary Process in the United States, Committee for Economic Development, New York, 1955

  1. Tax Incidence

Musgrave*, R. A., et al, “Distribution of Tax Payments by Income Groups,” National Tax Journal, March, 1951

Little, I. M. D., “Direct versus Indirect Taxes, Economic Journal, September, 1951

Musgrave*, R. A., “On Incidence,” Journal of Political Economy, August, 1953

Bach*, G. L., “The Impact of Moderate Inflation on Income and Assets of Economic Groups,” Federal Tax Policy for Economic Growth and Stability, Joint Committee on the Economic Report, Washington, D. C., November 9, 1955, p. 71

Musgrave*, R.A., “Incidence of the Tax Structure and its Effects on Consumption,” Federal Tax Policy for Economic Growth and Stability, Joint Committee on the Economic Report, Washington, D. C., November 9, 1955, p. 96

  1. Cyclical Aspects

Slichter*, S. H., “The Economics of Public Works,” American Economics Review, 1934, and Readings in Fiscal Policy

Lutz, H. L., “Federal Depression Financing and its Consequences,” Harvard Business Review, 1938, and Readings

Myrdal* G., “Fiscal Policy in the Business Trade,” American Economic Review Supplement, 1939, and Readings

Hagen, E. E., “Timing and Administering Fiscal Policy,” American Economic Review, May, 1948

Committee on Public Issues of the American Economic Association*, “The Problem of Economic Instability,” American Economic Review, 1950, and Readings

Smithies*, A., “The American Economic Association Committee Report on Economic Instability,” American Economic Review, 1951, and Readings

Phillips, A. W., “Stabilization Policy in a Closed Economy,” The Economic Journal, June, 1954, pp. 290-323

Committee for Economic Development*, Problems in Anti-Recession Policy, September 1954

  1. Alternative Budgets for Full Employment

Kaldor*, N., Appendix C in W. H. Beveridge, Full Employment in a Free Society, 1945

Musgrave*, R. A., “Alternative Budget Policies for Full Employment,” American Economic Review, 1945, and Readings

Musgrave*, R. A. and M. H. Miller, “Built-In Flexibility,” American Economic Review, 1948 and Readings

Bishop*, R. L., “Alternative Expansionist Policies,” Income, Employment and Public Policy: Essays in Honor of Alvin H. Hansen, New York, 1948

Stein, H., “Budget Policy to Maintain Stability,” Problems in Anti-Recession Policy, Committee for Economic Development, September, 1954

Hagen*, E. E., “Federal Taxation and Economic Stabilization,” Federal Tax Policy for Economic Growth and Stability, Joint Committee on the Economic Report, November 9, 1955, pp. 58-70

Lusher, D. W., “The Stabilizing Effectiveness of Budget Flexibility,” Policies to Combat Depression, a conference of the Universities-National Bureau Committee for Economic Research, Princeton, 1956

  1. Balanced Budget Multiplier

Wallich*, H. C., “Income-Generating Effects of a Balanced Budget,” Quarterly Journal of Economics, November, 1944

Haavelmo*, T., “Multiplier Effects of a Balanced Budget,” Econometrica, 1945, and Readings

Haberler, G., “Multiplier Effects of a Balanced Budget,” Econometrica, April, 1946

Baumol, W. J. and M. H. Preston, “More on the Multiplier Effects of a Balanced Budget under Full Employment,” American Economic Review, March, 1955

  1. The National Debt

Studenski, P., “The Limits of Possible Debt Burdens—Federal, State, and Local,” American Economic Review, Supplement, 1937

Haley*, R. F., “The Federal Budget: Economic Consequences of Deficit Financing,” American Economic Review, 1941, and Readings

Williams*, H. H., “Deficit Spending,” American Economic Review, February, 1941 and Postwar Monetary Plans and other Essays, 1944

Ratchford, B. U., “The Burden of a Domestic Debt,” American Economic Review, 1942, and Readings

Williams, J. H., “The Implications of Fiscal Policy for Monetary Policy and the Banking System,” Proceedings of the American Economic Association, 1942, and Readings

Domar*, E. D., “The ‘Burden of the Debt’ and the National Income,” American Economic Review, 1944, and Readings

Simons*, H., “On Debt Policy,” Journal of Political Economy, 1944, and Readings

Seltzer, L. H., “Is a Rise in Interest Rates Desirable or Inevitable?” American Economic Review, 1945, and Readings

Wallich, H. C., “Debt Management as an Instrument of Economic Policy,” American Economic Review, June, 1946

Roosa, R. V., “Integrating Debt Management and Open Market Operations,” American Economic Review, 1952, and Readings

Burkhead*, J., “The Balanced Budget,” Quarterly Journal of Economic, 1954, and Readings

  1. Inflation and War Finance

Sprague, O. M. W., “Loans and Taxes in War Finance,” American Economic Review, Proceedings, 1917, and Readings

Keynes*, J. M., How to Pay for the War, London, 1940

Smithies*, A., “The Behavior of Money National Income under Inflationary Conditions,” Quarterly Journal of Economics, 1942, and Readings

Fellner*, W. J., “Postscript on War Inflation: A Lesson from World War II,” American Economic Review, 1947, and Readings

Fetter*, F., “The Economic Reports of the President and the Problem of Inflation,” Quarterly Journal of Economics, 1949, and Readings

Wald, H. P., “Fiscal Policy, Military Preparedness, and Postwar Inflation,” National Tax Journal, 1949, and Readings

Hart, A. G., Defense Without Inflation, New York, 1951

  1. Effect on Incentives: Incentive Taxation

Domar*, E. D., and R. A. Musgrave, “Proportional Income Taxation and Risk Taking,” Quarterly Journal of Economic, May, 1954

Butters, J. K., and J. Lintner, Effect of Federal Taxes on Growing Enterprises, Boston, 1945

Groves*, H. M., Postwar Taxation and Economic Progress, New York, 1946, Chapter 11

Shelton, J. P., and G. Ohlin, “A Swedish Tax Provision for Stabilizing Business Investment,” American Economic Review, June, 1952

Brown*, R. S., “Techniques for Influencing Private Investment,” Income Stabilization in a Developing Democracy, M. Millikan, ed., 1953, pp. 416-432

Domar*, E. D., “The Case for Accelerated Depreciation,” Quarterly Journal of Economics, February, 1953

Butters*, J. K., “Taxation, Incentives, and Financial Capacity,” American Economic Review, Supplement, 1954, and Readings

Brown, E. C., “The New Depreciation Policy under the Income Tax: An Economic Analysis,” National Tax Journal, March, 1955

Goode*, R., “Accelerated Depreciation Allowances as a Stimulus to Investment,” Quarterly Journal of Economics, May, 1955

Break*, G. F., “Effects of Taxation on Work Incentives,” Federal Tax Policy for Economic Growth and Stability, Joint Committee on the Economic Report, Washington, D. C., November 9, 1955, p. 192

Brown*, E. C., “Weaknesses of Accelerated Depreciation as an Investment Stimulus,” Federal Tax Policy for Economic Growth and Stability, Joint Committee on the Economic Report, Washington, D. C., November 9, 1955, p. 495

Butters*, J. K., “Effects of Taxation on the Investment Capacities and Policies of Individuals,” Federal Tax Policy for Economic Growth and Stability, Joint Committee on the Economic Report, Washington, D. C., November 9, 1955, p. 126

Greenewalt*, C. H., “Effect of High Tax Rates on Executive Incentive,” Federal Tax Policy for Economic Growth and Stability, Joint Committee on the Economic Report, Washington, D. C., November 9, 1955, p. 185

Long*, C. D., “Impact of Federal Income Tax on Labor Force Participation,” Federal Tax Policy for Economic Growth and Stability, Joint Committee on the Economic Report, Washington, D. C., November 9, 1955, p. 153

Kaldor*, N., “An Expenditure Tax,” London, 1955

  1. Particular Taxes

Simons, H., Personal Income Taxation, Chicago, 1938

Brown*, E. C., “Analysis of Consumption Taxes in Terms of the Theory of Income Determination,” American Economic Review, March, 1950

Goode*, R., Corporation Income Tax, New York, 1951

Royal Commission on the Taxation of Profits and Income, First Report, February, 1953; Second Report, April, 1954; Final Report, June, 1955

Due, J. F., “Economics of Commodity Taxation and the Present Excise Tax System,” Federal Tax Policy for Economic Growth and Stability, Joint Committee on the Economic Report, Washington, D. C., November 9, 1955, p. 547

Keith*, G., “Economic Impact of the Corporation Income Tax,” Federal Tax Policy for Economic Growth and Stability, Joint Committee on the Economic Report, Washington, D. C., November 9, 1955, p. 658

Goode, R., “The Corporate Income Tax in a Depression,” Policies to Combat Depression, a conference of the Universities-National Bureau Committee for Economic Research, 1956

Merriam, I. C., “Social Security Programs and Economic Stability,” Policies to Combat Depression

Pechman, J. A., “Yield of the Individual Income Tax During A Recession,” Policies to Combat Depression

  1. Inter-Governmental Fiscal Relations

Maxwell*, J. A., “Intergovernmental Fiscal Devices for Economic Stabilization,” Federal Tax Policy for Economic Growth and Stability, Joint Committee on the Economic Report, Washington, D. C., November 9, 1955, p. 807

Heer*, C., “Stabilizing State and Local Finance,” Policies to Combat Depression, 1956

U. S. Treasury Department, Committee on Inter-Governmental Fiscal Relations, Federal, State, and Local Government Fiscal Relations, 78th Congress, 1st Session, Senate Document No. 69, 1943

U. S. Bureau of the Census, Compendium of State Government Finances (an annual series)

Same source, Compendium of City Government Finances (an annual series)

Tax Institute, Federal-State-Local Tax Correlation (A Symposium), December, 1953

The Council of State Governments, Federal Grants-in-Aid, 1949

  1. Growth and Economic Development

Bernstein*, E. M. and I. G. Patel, “Inflation in Relation to Economic Development,” International Monetary Fund, Staff papers, II, 1951-52

United Nations*: Fiscal Division, “Taxation and Economic Development in Asian Countries,” Economic Bulletin for Asia and the Far East, Vol. IV, November, 1953

Gurley, J. A., “Fiscal Policy in a Growing Economy,” Journal of Political Economy, December, 1953

Papers and Proceedings of the Conference on Agricultural Taxation and Economic Development, H. P. Wald, and J. N. Froomkin, eds., Cambridge, Massachusetts, 1954 (Harvard University Law School, International Program in Taxation)

  1. Special Problems

Clark*, C., “Public Finance and Changes in the Value of Money,” The Economic Journal, December, 1945

Clark*, C., “The Danger Point in Taxes,” Harper’s Magazine, December, 1950

Goode*, R., “An Economic Limit on Taxes: Some Recent Discussion,” National Tax Journal, September, 1952

Caplan, B., “A Case Study: The 1948-1949 Recession,” Policies to Combat Depression, 1956

Fox, K. A., “The Contribution of Farm Price Support Programs to General Economic Stability,” Policies to Combat Depression, 1956

Gordon, R. A., “Types of Depressions and Programs to Combat Them,” Policies to Combat Depression

Grebler, L., “Housing Policies to Comat Depression,” Policies to Combat Depression

Johnson, D. G., “Stabilization of International Commodity Prices,” Policies to Combat Depression

Owen, W., “Self-Liquidating Public Works to Combat Depression,” Policies to Combat Depression

Source: Duke University, David M. Rubenstein Rare Book and Manuscript Library. Economists’ Papers Archive. Box 17, Folder “Fiscal and Monetary Policy”.

___________________________

FINAL EXAMINATION
14.472 Fiscal Policy
Monday, May 20, 1957

E. D. Domar

ANSWER ALL QUESTIONS. THE QUALITY OF YOUR REASONING IS THE MOST IMPORTANT PART OF YOUR ANSWERS.

  1. [35%] Compare and contrast monetary and fiscal policies as methods of achieving a steadily expanding economy (without inflation or depression). Include, but don’t limit yourself to, the following points:
      1. The theoretical foundation of each.
      2. Methods used.
      3. Effects on distribution of income and wealth.
      4. Social and political repercussions of each.
      5. The effectiveness and limitations of each.

Do they overlap? Can you work out a synthesis?

  1. [20%] “Government spending tends to be like a drug, in that it takes larger and larger doses to get results, and all the time debt and taxes get higher and higher.”
    Analyze this statement and comment as fully as you can. Compare the effect of government expenditures with that of private.
  2. [15%] “The best cure against inflation is increased production.”
    Analyze this statement and comment on it. Include in your comments the monetary and fiscal implications of this statement.
  3. [15%] What are the so-called “Built-in-Stabilizers?” Discuss fully and indicate how they operate in (a) depression and (b) inflation.
  4. [15%] “The purpose of taxation is never to raise money but to leave less in the hands of the taxpayer.”
    Comment fully and indicate the limitations of this statement. Can you identify the author? (No great penalty if you cannot.)

Source: Duke University, David M. Rubenstein Rare Book and Manuscript Library. Economists’ Papers Archive. Box 16, Folder “Examination. Public Finance and Fiscal Policy”.

Notes on Final Exam:

Question II comes from a review of Stuart Chase, Where’s the Money Coming From? Published in the Monthly Bulletin of the National City Bank of New York that I was fortunate to find inserted into the Congressional Record Volume 93—Part 4 (May 8, 1947, p. 4827);

Question III. Domar liked this question enough to have used it at least twice. See January 23, 1958 Exam at Johns Hopkins; January 26, 1966 at M.I.T.;

Question V. The sentence quoted comes from Abba Lerner’s The Economics of Control, p. 307.

___________________________

Image Source: Evsey D. Domar at the MIT Museum legacy website.

 

Categories
Chicago Economic History Suggested Reading Syllabus

Chicago. Reading Assignments for Development of Monetary and Financial Institutions. Hamilton, 1952

 

The 1960 reading assignments for Earl J. Hamilton’s course on the historical development of monetary and financial institutions at the University of Chicago were transcribed and posted earlier. It turns out that a copy of the 1952 reading assignments for the course can be found filed in a folder in the Milton Friedman Papers at the Hoover Institution. Since there was no name on the 1952 reading list, an archivist is likely to have erroneously assumed that the course had been taught by Friedman and so filed it with other materials from those courses taught by Milton Friedman. A casual comparison of the two sets of reading assignments is enough to verify that bibliographic formatting (e.g., Library of Congress call numbers are included for all items) and the overwhelming majority of items are common to the two lists (with some shuffling in the order of the readings). 

____________________________

Course Description (1951)

[Economics] 334. The Development of Monetary and Financial Institutions. The evolution of monetary and banking systems in leading nations, the rise of organized dealings in foreign exchange, the emergence of great money markets, the origin and growth of national debt and the economic consequences of inflation and deflation. Theories underlying and reflecting monetary and financial changes. Prereq: Econ 222 or 230. Spr: MTuWF 9:30; Hamilton

Source: University of Chicago. Announcements, Vol. 50, No. 9 (July 20, 1950). The Division of the Social Sciences, Sessions of 1950-1951, p. 30.

____________________________

Course Reading Assignments

Economics 334

Assignments to be read before July 16, 1952

  1. Luigi Einaudi, “The Medieval Practice of Managed Currency,” in A.D. Gayer (Editor), The Lessons of Monetary Experience, pp. 259-268. HG 255.L63
  2. W. C. Mitchell, “The Role of Money in Economic Theory,” in The Backward Art of Spending Money, Pp. 149-176. HB 33.M68.
  3. Official Papers by Alfred Marshall, pp. 3-16. HG171.M318.
  4. R. G. Hawtrey, Currency and Credit, Chapter on “The Bank Restriction of 1797.” HG221.H4
  5. J. M. Keynes, A Tract on Monetary Reform, Chapters I-II, IV. HG221.K4
  6. J. M. Keynes, A Treatise on Money, Vol. II, Chapter 30. HG221.K422
  7. Alfred Marshall, Money, Credit, and Commerce, Book IV, and Appendix A. HG221.M35
  8. Charles F. Dunbar (Revised and edited by O. M. W. Sprague), The Theory and History of Banking, Chapters VIII (“The English Banking System”), IX (“The French Banking System”), X (“The German Banking System”), XI (“The National Banks of the United States”). HG1586.D9
  9. Earl J. Hamilton, American Treasure and the Price Revolution in Spain, Chapter XIII. H31.H33, v. 43.
  10. *Earl J. Hamilton, “The Foundation of the Bank of Spain,” Journal of Political Economy, Vol. LIII (1945), pp. 97-114. HB1.J7.
  11. Earl J. Hamilton, “Prices and Wages at Paris under John Law’s System,” Quarterly Journal of Economics, Vol. LI, (1936-1937), pp. 42-70. HB1.Q3
  12. Walter Bagehot, Lombard Street. HG3000.L82B3
  13. R. S. Sayers, “The Question of the Standard in the Eighteen-Fifties,” Economic History (a supplement to the Economic Journal), Vol. II, pp. 575-601. HB1.E31
  14. Rufus S. Tucker, “The Myth of 1849,” in C.O Hardy, Is There Enough Gold? Appendix A, pp. 177-199. HG289.H28
  15. Davis R. Dewey, Financial History of the United States, 12th Ed. Pp. 320-328 (“Arguments in Favor of a National Banking System” and “National Banking Act of 1863.”) HJ241.D576
  16. *R. G. Hawtrey, A Century of Bank Rate, Chapters I-IV. HG1623.G7H4

*Read for interpretation and point of view. Do not try to remember facts.

There will be an examination on July 16 on these assignments and the lectures.

*  *  *  *  *  *  *  *  *  *  *  *  *  *

Economics 334

Assignments to be read by not later than August 13, 1952

  1. J. Silberling, “Financial and Monetary Policy of Great Britain during the Napoleonic Wars,” Quarterly Journal of Economics, Vol. XXXVIII (1923-24), pp. 214-33, 397-439. HB1.Q3, v.38
  2. H. Clapham, The Bank of England, Vol. II, Chapters VI-VIII and Epilogue. HG2996.C6
  3. M. Keynes, Essays in Persuasion, Part II, Chaps. 1 and 3; Part III, Chapter 5; Part V, Chapter 2. In the London, 1933 edition these chapters cover pages 77-79, 105-17, 244-70, 358-73. HC57.K471
  4. H. Robertson, Essays in Monetary Theory, Chaps. I and XII. HB171.R544
  5. Fred H. Klopstock, “Monetary Reform in Western Germany,” Journal of Political Economy, August, 1949. HB1.J7, v. 57
  6. Alfred Marshall, Money, Credit, and Commerce, Book II. HG221.M35
  7. Henri Hauser, Les origins historiques des problèmes économiques actuels, Paris, 1930, pp. 70-90.
  8. M. W. Sprague, Crises under the National Banking System, Washington, 1910, pp. 1-107. HB3743.S7
  9. M. Keynes, A Treatise on Money, Vol. II, Chaps. 35 and 37. HG221.K422
  10. M. Keynes, The General Theory of Employment, Interest and Money, Chapter 23. HB171.K46
  11. W. Kemmerer, The ABC of the Federal Reserve System, Princeton University Press, 1936, Chapters I-V. HG2563.K31
  12. Jacob Viner, Studies in the Theory of International Trade, Chapters III-V. HF1007.V75
  13. Adam Smith, Wealth of Nations, “Digression concerning Banks of Deposit, particularly concerning that of Amsterdam,” in Book IV, Chapter III, Part I. HB161.S65.
  14. Knut Wicksell, “The Influence of the Rate of Interest on Prices,” Economic Journal, Vol. XVII (1907), pp. 213-220. YW16 (reprint)

There will be an hour examination on August 13th.

Source: Hoover Institution Archives. Papers of Milton Friedman. Box 78, Folder 1 “University of Chicago Econ 334”.

Image Source:  University of Chicago Photographic Archive, apf1-02446, Special Collections Research Center, University of Chicago Library.

 

Categories
Economic History Exam Questions Harvard

Harvard. Final exams for European and U.S. Economic History. Gay and Klein, 1911

This post adds  final examinations for the 1910-11 academic year to previous posts dedicated to two economic history courses taught by Edwin F. Gay during the second decades of the twentieth century at Harvard. 

_____________________

Instructors: Edwin F. Gay, Julius Klein

Biographical information

_____________________

Reading list: First term, 1910-11.
Economics 6a.

European Industry and Commerce in the Nineteenth Century.

Final exam. First term, 1910-11.
Economics 6a.

[European Industry and Commerce in the Nineteenth Century]

  1. (a) “The essence of the industrial revolution is the substitution of competition for the mediaeval regulations which had previously controlled the production and distribution of wealth….Competition came to be believed in as a gospel.” (Toynbee.)
    Give illustrations of the substitution of competition, and state why, in your opinion, the good rather than the evil of competition was emphasized at this period.
    (b) Toynbee said: “The effects of the industrial revolution prove that free competition may produce wealth without producing well-being.” A Boston reformer, speaking of the advent of machinery, says, “The profits from the machine were absorbed by capital,” so that the people did not have their share. Does the economic history of the nineteenth century support these views?
  2. (a) Trace concisely the influence of agricultural interests upon the tariff history of England, France and Germany in the nineteenth century.
    (b) Has England been more affected by the agricultural depression of the last thirty years than other European countries? Why has it not been more influenced by the “protectionist reaction?”
  3. How has the French railroad policy differed from that of Prussia? Which has been the more beneficial? Give reasons.
  4. Do the actual conditions warrant the assertions concerning “the ominous situation of British trade?” Give reasons.
  5. State succinctly the chief facts concerning:—
    (a) Assignats.
    (b) British shipping subsidies.
    (c) Crisis of 1857.

Source: Harvard University Archives, Examination papers, Mid-years (HUC 7000.55). Box 8, Examination Papers, Mid-years, 1910-11.

cf. Final exam. First term, 1914-15.
Economics 2a.

European Industry and Commerce in the Nineteenth Century.

_____________________

Reading list: Second term, 1910-11.
Economics 6b.

Economic and Financial History of the United States.

 

Final exam: Second term, 1910-11.
Economics 6b.

[Economic and Financial History of the United States]

  1. Outline fully the topics you would discuss if you were to write a thesis on the history of the iron and steel industry in the United States.
  2. Comment on the following:—
    1. “We import annually millions of dollars’ worth of tropical products that could be grown in the United States.” (Report of Secretary of Agriculture for 1901.)
    2. “The sum paid by American producers and manufacturers to these foreign bottoms during the past year was $500,000,000, a sum sufficient to dig the Panama Canal and operate it for twenty years. Remember that all this good American money has gone into the pockets of foreigners.” (Admiral Evans in a recent magazine article.)
  3. (a) Outline our tariff history since 1890.
    (b) Comment on Senator Aldrich’s statement: “I do not believe there are any duties levied in this bill [the tariff act of 1909] that are excessive or are prohibitory.”
    (c) The American experience with reciprocity. Are you in favor of the reciprocity treaty with Canada? State your reasons.
  4. Rhodes in his history of the United States says: “This tendency [the accumulation of large fortunes and the development of abject poverty] had begun before the War and has been the result rather of the constantly deteriorating character of the European immigration than of industrial changes on our own soil.”
    Do you agree with this view? Give your reasons.
  5. Comment on the following: “The year 1896 was in fact one of those periods rare in the history of any country of which it could be said that a given chapter had definitely closed and that another was about to open.”
  6. Compare briefly the conditions before and after the Civil War in respect to
    1. The defects of our banking system;
    2. The public land policy;
    3. Governmental assistance to and control of transportation enterprise.

Source: Papers set for Final Examinations in History, Government, Economics, …, Landscape Architecture, Music in Harvard College. June 1911, p. 44. In Harvard University Archives, Examination papers, 1873-1915 (HUC 7000.25). Box 9. Examination Papers, 1910-11.

cf. Final Exam. Second term, 1914-15.

Economics 2b
Economic and Financial History of the United States
.

Image Source: Edwin Francis Gay and Julius Klein, respectively, from The World’s Work, Vol. XXVII, No. 5 (March 1914) and Harvard Album 1920.

 

Categories
Exam Questions Harvard Socialism Suggested Reading Syllabus Undergraduate

Harvard. Readings and Final Exam for Normative Aspects of Economic Policy. Bergson, 1960

 

The reading list and final exam questions from 1959 for Abram Bergson‘s Harvard undergraduate course “Normative Aspects of Economic Policy” have been posted earlier. This post provides material for the same course taught in the spring term of 1960. The reading lists are completely identical, but this time I have gone to the trouble of providing links to most of the course readings.  The exam questions for the 1960 do indeed differ from those of 1959 while covering broadly the same material.

_________________

Enrollment

[Economics] 111a. Normative Aspects of Economic Policy. Professor Bergson. Half course. (Spring)

Total 36: 3 Graduates, 14 Seniors, 7 Juniors, 7 Sophomores, 1 Freshman, 3 Radcliffe, 1 Other.

Source: Harvard University. Report of the President of Harvard College, 1959-1960, p. 82.

_________________

HARVARD UNIVERSITY
Department of Economics

Economics 111a
Normative Aspects of Economic Policy
Spring Term: 1959-60

  1. The concept of economic efficiency.

Scitovsky, Welfare and Competition, Chicago, 1951, Chapter I.

  1. Consumers’ goods distribution and labor recruitment: the efficiency of perfect competition: other forms of market organization.

Scitovsky, Chapters II-V, XVI (pp. 338-41), XVIII, XX (pp. 423-427).
A. P. Lerner, Economics of Control, New York, 1946, Chapter 2.

  1. Conditions for efficiency in production.

Scitovsky, Chapters VI-VIII.
Lerner, Chapter 5.

  1. Production efficiency under perfect competition; monopolistic markets.

See the readings under topic 3.
Scitovsky, Chapter X, XI, XII, XV, XVI (pp. 341-363), XVII, XX (pp. 428-439).
Lerner, Chapters 6, 7.

  1. The optimum rate of investment.

Scitovsky, Chapter IX (pp. 216-228).
A. C. Pigou, Economics of Welfare, fourth ed., London, 1948, pp. 23-30.”Wa

  1. Price policy for a public enterprise.

Lerner, Chapter 15.
I. M. D. Little, A Critique of Welfare Economics, 2nd ed., Oxford, 1957, Chapter XI.
O. Eckstein, Water Resource Development, Cambridge, 1958, pp. 47-70, pp. 81-109.

  1. Socialist economic calculation.

O. Lange, On the Economic Theory of Socialism, Minn., 1938, pp. 55-141.
F. Hayek, Socialist Calculation: Economica, May 1940
A. Bergson, Socialist Economics, in H. Ellis, ed., A Survey of Contemporary Economics, Philadelphia, 1948.
M. Dobb, Economic Theory and Socialism, New York, 1955, pp. 41-92.

  1. Economic calculation in underdeveloped countries.

A. Datta, Welfare versus Growth Economics, Indian Economic Journal, October 1956.
T. Scitovsky, Two Concepts of External Economics, Journal of Political Economy, April 1954.
J. Tinbergen, The Design of Development, Balto., Md., 1958.

  1. The concept of social welfare.

The writings of Bergson and Dobb under topic 7.
Pigou, Economics of Welfare, Chapters I, VIII.
Lerner, Chapter 3.
J. R. Hicks, Foundations of Welfare Economics, Economic Journal, December 1939.
Arthur Smithies, Economic Welfare and Policy, in A. Smithies et al., Economics and Public Policy, Washington, 1955.

 

Other References on the Concept of Social Welfare and Optimum Conditions

M. W. Reder, Studies in the Theory of Welfare Economics, New York 1947.

P. A. Samuelson, Foundations of Economic Analysis, Cambridge, 1947, Chapter VIII.

K. Boulding, Welfare Economics, in B. Haley, A Survey of Contemporary Economics, Homewood, Illinois, 1952.

H. Myint, Theories of Welfare Economics, Cambridge, Mass., 1948.

J. A. Hobson, Work and Wealth, London, 1933.

J. M. Clark, Guideposts in Time of Change, New York, 1949.

J. de V. Graaf, Theoretical Welfare Economics, Cambridge, 1957.

F. M. Bator, The Simple Analytics of Welfare Maximization, American Economic Review, March 1957.

A. Bergson, A Reformulation [of Certain Aspects] of Welfare Economics, Quarterly Journal of Economics, February 1938.

P. A. Samuelson, Evaluation of Real National Income, Oxford Economic Papers, January 1950.

A. C. Pigou, Some Aspects of Welfare Economics, American Economic Review, June 1951.

T. Scitovsky, The State of Welfare Economics, American Economic Review, June 1951.

J. E. Meade, Trade and Welfare, New York, 1955, Part I.

[Note: no additional assignment for the reading period]

Source:  Harvard University Archives. Syllabi, course outlines and reading lists in Economics, 1895-2003. Box 7, Folder “Economics, 1959-60”.

_________________

HARVARD UNIVERSITY
Department of Economics

Economics 111a
Final Examination

June 2, 1960

Answer four and only four of the following six questions.

  1. Explain the “price-consumption” curve for a single household in a perfectly competitive consumers’ goods market. What determines the shape of the curve? By use of this curve, show how the household’s consumption might be affected by a percentage sales tax on one commodity. What determines the total taxes paid by the household?
  2. In an economy which otherwise is perfectly competitive, a trade union arbitrarily limits entry of workers into a single industry. In equilibrium, what conditions for an economic optimum are violated?
  3. “Under ‘free’ competition it is true that individual firms have monopoly power and hence charge prices above marginal costs. But since there is free entry, there hardly can be any serious economic waste on this account, for prices cannot long exceed average cost.” Discuss.
  4. For purposes of fixing prices for a public enterprise, what arguments might be advanced for and against acceptance of each of the following theoretic principles:
    1. Maximization of profits;
    2. Pricing at average cost, including a “normal” competitive return on invested capital;
    3. Pricing at marginal cost;
    4. Pricing at minimum average costs.
  5. Explain briefly:
    1. Parametric function of prices;
    2. “Technological” versus “pecuniary” external economies;
    3. “Accounting prices” in economics of development;
    4. “Defective telescopic faculty.”
  6. Discuss the different approaches employed in welfare economics to the problem of income distribution.

 

Source:  Harvard University Archives. Papers Printed for Final Examinations: History, History of Religions, …, Economics, …,Naval Science, Air Science. June 1960. In Social Sciences, Final Examinations, June 1960 (HUC 2000.28, No. 128).

Portrait of Abram Bergson. See Paul A. Samuelson, “Abram Bergson, 1914-2003: A Biographical Memoir”, in National Academy of Sciences, Biographical Memoirs, Volume 84 (Washington, D.C.: 2004).